Difference between revisions of "2018 AMC 8 Problems/Problem 2"

(Problem 2)
Line 1: Line 1:
==Problem 2==
+
==Problem 24==
What is the value of the product<cmath>\left(1+\frac{1}{1}\right)\cdot\left(1+\frac{1}{2}\right)\cdot\left(1+\frac{1}{3}\right)\cdot\left(1+\frac{1}{4}\right)\cdot\left(1+\frac{1}{5}\right)\cdot\left(1+\frac{1}{6}\right)?</cmath>
 
 
 
<math>\textbf{(A) }\frac{7}{6}\qquad\textbf{(B) }\frac{4}{3}\qquad\textbf{(C) }\frac{7}{2}\qquad\textbf{(D) }7\qquad\textbf{(E) }8</math>
 
  
 
==Solution==
 
==Solution==
 
You may simply the expression to get <math>\frac{2}{1} \cdot \frac{3}{2} \cdot \frac{4}{3} \cdot \frac{5}{4} \cdot \frac{6}{5} \cdot \frac{7}{6}</math>.
 
 
Using telescoping, many things cancel out. You are left with <math>\frac{7}{1}</math> which is equivelant to <math>7</math>, or <math>\textbf{(D) }</math>
 
  
 
==See Also==
 
==See Also==
{{AMC8 box|year=2018|num-b=1|num-a=3}}
+
{{AMC8 box|year=2018|num-b=23|num-a=25}}
  
 
{{MAA Notice}}
 
{{MAA Notice}}

Revision as of 16:10, 21 November 2018

Problem 24

Solution

See Also

2018 AMC 8 (ProblemsAnswer KeyResources)
Preceded by
Problem 23
Followed by
Problem 25
1 2 3 4 5 6 7 8 9 10 11 12 13 14 15 16 17 18 19 20 21 22 23 24 25
All AJHSME/AMC 8 Problems and Solutions

The problems on this page are copyrighted by the Mathematical Association of America's American Mathematics Competitions. AMC logo.png